Distributions/ Ratios
Hi, I often have a a hard time determining when I need to make deductions of ratios like in t...
Rosibeth23 on January 4 at 12:55AM
  • October 2010 LSAT
  • SEC2
  • Q18
2
Replies
Why is option D wrong?
Based on the set up, 1-2-1-2 = 6 slots and that does not violate any setup rules. M-W&Y-M-W&Y cou...
Vineyard on November 1 at 01:46PM
  • October 2010 LSAT
  • SEC2
  • Q19
2
Replies
Why did we change the set up to the game ?
Would it be more productive setting the days as our base ? I did the actors, why did the tutors d...
Nativeguy on July 20 at 05:47PM
  • October 2010 LSAT
  • SEC2
  • Q6
2
Replies
W in quarter 4
Confused because three grants in quarter three allows for both W's inclusion and W's exclusion fr...
farnoushsalimian on October 19, 2022
  • October 2010 LSAT
  • SEC2
  • Q23
2
Replies
Colors
I got stuck in the exact spot the LSAT wanted me to get stuck. I assumed, that all the colors had...
allisonfarley on January 5, 2021
  • October 2010 LSAT
  • SEC2
  • Q15
2
Replies
Setup
Hello, I don't seem to be grasping why exactly one of I/p has to be in. I realize that there is ...
SorooshKosha on March 19, 2020
  • October 2010 LSAT
  • SEC2
  • Q12
6
Replies
Video explanation
Can there be a video for the game? I feel like I missed a key deduction.
meganw on February 17, 2020
  • October 2010 LSAT
  • SEC2
  • Q1
2
Replies
If I get a question like this one, should I dra...
This is a general question but I know that I alway draw the diagram out quickly and look for ded...
MACZ on January 6, 2020
  • October 2010 LSAT
  • SEC2
  • Q15
2
Replies
Missing
I am missing something mentally on this, because going through I didn't have enough to set up. It...
Cal on May 30, 2019
  • October 2010 LSAT
  • SEC2
  • Q12
6
Replies
How to approach
Hello, I'm wondering if there is some strategy on how to approach this type of questions, where t...
Valerie on May 10, 2019
  • October 2010 LSAT
  • SEC2
  • Q5
8
Replies
Why are there no videos for this game?
I'd like to see how it is set up.
Jessica-Killeen on January 31, 2019
  • October 2010 LSAT
  • SEC2
  • Q2
6
Replies
Answer Choice A
I am a bit confused as to how answer choice A could be true. In the hypothetical, it is writte...
EmilyMarieMenendez on September 6, 2018
  • October 2010 LSAT
  • SEC2
  • Q20
4
Replies
Game Setup?
Can you please post the game set up for this one as well, I missed two questions that appeared in...
MGN2014 on October 21, 2014
  • October 2010 LSAT
  • SEC2
  • Q20
2
Replies
Clarify
How can choice A be true?
Melissasummer26 on October 16, 2014
  • October 2010 LSAT
  • SEC2
  • Q8
6
Replies
Game Setup?
Can you please post the game set up for these questions?
MGN2014 on September 26, 2014
  • October 2010 LSAT
  • SEC2
  • Q13
2
Replies